Difference between revisions of "1967 IMO Problems/Problem 1"

m
m
 
(19 intermediate revisions by the same user not shown)
Line 1: Line 1:
Let <math>ABCD</math> be a parallelogram with side lengths <math>AB = a</math>, <math>AD = 1</math> and with <math>\angle BAD = \alpha</math>.
+
Let <math>ABCD</math> be a parallelogram with side lengths <math>AB = a</math>, <math>AD = 1</math>, and with <math>\angle BAD = \alpha</math>.
 
If <math>\Delta ABD</math> is acute, prove that the four circles of radius <math>1</math> with centers <math>A</math>, <math>B</math>, <math>C</math>, <math>D</math> cover the parallelogram if and only if
 
If <math>\Delta ABD</math> is acute, prove that the four circles of radius <math>1</math> with centers <math>A</math>, <math>B</math>, <math>C</math>, <math>D</math> cover the parallelogram if and only if
  
<math>a\leq \cos \alpha+\sqrt{3}\sin \alpha</math> (1)
+
<math>a\leq \cos \alpha+\sqrt{3}\sin \alpha</math> <math>\ \ \ \ \ \ \ \ \ \ (1)</math>
  
 
----
 
----
 +
 
==Solution==
 
==Solution==
 
To start our proof we draw a parallelogram with the requested sides. We notice that by drawing the circles with centers A, B, C, D that the length of <math>a</math> must not exceed 2 (the radius for each circle) or the circles will not meet and thus not cover the parallelogram.
 
To start our proof we draw a parallelogram with the requested sides. We notice that by drawing the circles with centers A, B, C, D that the length of <math>a</math> must not exceed 2 (the radius for each circle) or the circles will not meet and thus not cover the parallelogram.
Line 30: Line 31:
  
 
--[[User:Bjarnidk|Bjarnidk]] 02:16, 17 May 2013 (EDT)
 
--[[User:Bjarnidk|Bjarnidk]] 02:16, 17 May 2013 (EDT)
 +
 +
 +
==Remarks (added by pf02, September 2024)==
 +
 +
<math>\mathbf{Remark\ 1}</math>. I am sorry to be so harshly critical, but the
 +
solution above is deeply flawed.  Not only it has errors, but the
 +
logic is flawed.
 +
 +
It shows that when <math>a = 2, \alpha = \frac{\pi}{3}</math> the parallelogram
 +
is covered by the circles of radius <math>1</math> centered at <math>A, B, C, D</math>, and
 +
the inequality in the problem is true.  (Even this is incomplete, while
 +
giving too many, unnecessary details.)  (Note that this is not a case
 +
which satisfies the conditions of the problem since <math>\triangle ABD</math> is
 +
right, not acute.)
 +
 +
In the last two lines it gives some reasoning about other values of
 +
<math>\alpha</math> which is incomprehensible to this reader.
 +
 +
In one short sentence: this is not a solution.
 +
 +
<math>\mathbf{Remark\ 2}</math>. The problem itself is mildly flawed.  To see this,
 +
denote <math>S1, S2</math> the following two statements:
 +
 +
S1: The parallelogram <math>ABCD</math> is covered by the four circles of radius
 +
<math>1</math> centered at <math>A, B, C, D</math>.
 +
 +
S2: We have <math>a \le \cos \alpha + \sqrt{3} \sin \alpha</math>.
 +
 +
The problem says that if <math>\triangle ABD</math> is acute, <math>S1</math> and <math>S2</math> are
 +
equivalent, i.e. they imply each other.
 +
 +
Notice that <math>S2</math> can be rewritten as
 +
<math>a \le 2 \cos \left( \alpha - \frac{\sqrt{\pi}}{3} \right)</math>.
 +
 +
Now notice that if <math>a \le 1</math> then S1 is obviously true.  See
 +
the picture below:
 +
 +
[[File:Prob_1967_1_fig1.png|300px]]
 +
 +
Also, notice that if <math>a \le 1</math> and
 +
<math>\alpha \in \left( 0, \frac{\sqrt{\pi}}{2} \right) </math>
 +
then <math>S2</math> is true as well.  Indeed
 +
<math>\left( \alpha - \frac{\sqrt{\pi}}{3} \right) \in
 +
\left( -\frac{\sqrt{\pi}}{3}, \frac{\sqrt{\pi}}{6} \right)</math>, so
 +
<math>\cos</math> is <math>> \frac{1}{2}</math> on this interval, so the right hand side
 +
of <math>S2</math> is <math>> 1 \ge a</math>.
 +
 +
We see that if <math>a \le 1</math> and <math>\triangle ABD</math> is acute, both <math>S1</math>
 +
and <math>S2</math> are true.  We can not say that one implies the other in
 +
the usual meaning of the word "imply": the two statements just
 +
happen to be both true.
 +
 +
If we take <math>a > 1</math> then the problem is a genuine problem, and
 +
there is something to prove.
 +
 +
<math>\mathbf{Remark\ 3}</math>. In the proofs I give below, we will see
 +
where we need that <math>\triangle ABD</math> is acute.  We will see that
 +
<math>\alpha < \frac{\pi}{2}</math> is needed for the  technicalities of
 +
the proof.  The fact that <math>\angle ADB</math> is acute will be needed
 +
at one crucial point in the proof.
 +
 +
In fact, it is possible to modify <math>S2</math> to a statement <math>S3</math>
 +
similar to <math>S2</math> so that <math>S1</math> and <math>S3</math> are equivalent without
 +
any assumption on <math>\alpha</math>.  I will not go into this, I will
 +
just give a hint:  Denote <math>\beta = \angle ABC</math>.  If <math>\alpha</math>
 +
is acute, <math>\beta</math> is obtuse, and we can easily reformulate
 +
<math>S2</math> in terms of <math>\beta</math>.
 +
 +
<math>\mathbf{Remark\ 4}</math>. Below, I will give two solutions.
 +
Solution 2 is one I carried out myself and relies on a
 +
straightforward computation.  Solution 3 (it happens to be
 +
similar to Solution 2) is inspired by an idea by feliz shown
 +
on the web page
 +
https://artofproblemsolving.com/community/c6h21154p137323
 +
The author calls their text a solution, but it is quite
 +
confused, so I would not call it a good solution.  The idea
 +
though is good and nice, and it yields a nice solution.
 +
 +
 +
==Solution 2==
 +
 +
We can assume <math>a > 1</math>.  Indeed, refer to Remark 2 above to
 +
see that if <math>a \le 1</math> there is nothing to prove.
 +
 +
Note that instead of the statement <math>S1</math> we can consider the
 +
following statement <math>S1'</math>:
 +
 +
<math>S1'</math>: the circles of radius <math>1</math> centered at <math>A, B, D</math> cover
 +
<math>\triangle ABD</math>.
 +
 +
This is equivalent to <math>S1</math> because of the symmetry between
 +
<math>\triangle ABD</math> and <math>\triangle BCD</math>.
 +
 +
Let <math>F</math> be the intersection above <math>AB</math> of the circles of radius
 +
<math>1</math> centered at <math>A, B</math>.  The three circles cover <math>\triangle ABD</math>
 +
if an only if <math>F</math> is inside the circle of radius 1 centered at
 +
<math>D</math>.
 +
 +
[[File:Prob_1967_1_fig2.png|300px]]
 +
 +
This needs an explanation: Let <math>H</math> be the midpoint of <math>BD</math>, and
 +
consider <math>\triangle FHD</math>.  All the vertices of this triangle are
 +
in the circle centered at <math>D</math>, so the whole triangle is inside
 +
this circle.  It is obvious that <math>\triangle FHB</math> is inside the
 +
circle centered at <math>B</math>, and that <math>\triangle FAD, \triangle FAB</math>
 +
are inside the circles centered at <math>A, B</math>.
 +
 +
We will now show that <math>F</math> is inside the circle of radius 1 centered
 +
at <math>D</math> if an only if <math>DF \le 1</math>.
 +
 +
The plan is to calculate <math>DF</math> in terms of <math>a, \alpha</math> and impose
 +
this condition.  Let <math>FG \perp AB</math>, <math>DE \perp AB</math> and
 +
<math>FF' \parallel GE</math>.  From the right triangle <math>\triangle AFG</math> we
 +
have <math>FG = \sqrt{1 - \left( \frac{a}{2} \right)^2} =
 +
\frac{\sqrt{4 - a^2}}{2}</math>.  From the right triangle <math>\triangle
 +
DFF'</math> we have
 +
 +
<math>DF = \sqrt{(DF')^2 + (FF')^2} = \sqrt{(DE - FG)^2 + (AG - AE)^2} =
 +
\sqrt{\left( \sin \alpha - \frac{\sqrt{4 - a^2}}{2} \right)^2 +
 +
\left( \frac{a}{2} - \cos \alpha \right)^2}</math>
 +
 +
(Note that here we used the fact that <math>\alpha</math> is acute.  These
 +
equalities would look slightly differently otherwise.)
 +
 +
Now look at the condition <math>DF \le 1</math>, or equivalently <math>DF^2 \le 1</math>.
 +
Making all the computations and simplifications, we have
 +
<math>\sqrt{4 - a^2} \sin \alpha \ge 1 - a \cos \alpha</math>.
 +
 +
Now I would like to square both sides.  In order to get an
 +
equivalent inequality, we need to know that <math>1 - a \cos \alpha \ge 0</math>.
 +
This follows from the fact that <math>\angle ADB</math> is acute.  Indeed,
 +
denote <math>\angle ADB = \beta</math>.  From the law of sines in
 +
<math>\triangle ADB</math> we have
 +
<math>\frac{1}{\sin \angle ABD} = \frac{a}{\sin \beta}</math>.  Successively
 +
this becomes
 +
<math>\frac{1}{\sin (\pi - \alpha - \beta)} = \frac{a}{\sin \beta}</math>
 +
or <math>\frac{1}{\sin (\alpha + \beta)} = \frac{a}{\sin \beta}</math>
 +
or <math>(1 - a \cos \alpha) \sin \beta = a \sin \alpha \cos \beta</math>.
 +
From here we see that <math>\beta < \frac{\pi}{2}</math> implies the right
 +
hand side is positive, so <math>1 - a \cos \alpha > 0</math>.
 +
 +
Going back to our inequality, we can square both sides, and
 +
after rearranging terms we get that <math>DF \le 1</math> if and only if
 +
 +
<math>a^2 - 2a \cos \alpha + (1 - 4 \sin^2 \alpha) \le 0</math>.
 +
 +
View this as an equation of degree <math>2</math> in <math>a</math>.  The value of the
 +
polynomial in <math>a</math> is <math>\le 0</math> when <math>a</math> is between its solutions,
 +
that is
 +
 +
<math>\cos \alpha - \sqrt{3} \sin \alpha \le a \le
 +
\cos \alpha + \sqrt{3} \sin \alpha</math>.
 +
 +
Note that <math>\cos \alpha - \sqrt{3} \sin \alpha =
 +
2 \cos \left( \alpha + \frac{\pi}{3} \right)</math>.  If
 +
<math>\alpha \in \left( 0, \frac{\pi}{2} \right)</math> then
 +
<math>\left( \alpha + \frac{\pi}{3} \right) \in
 +
\left( \frac{\pi}{3}, \frac{5\pi}{6} \right)</math>,
 +
and it follows that
 +
<math>2 \cos \left( \alpha + \frac{\pi}{3} \right) \le 1</math>.
 +
 +
On the other hand, remember that we are in the case <math>a > 1</math>,
 +
so the left inequality is always true.  It follows that
 +
 +
<math>DF \le 1</math> (i.e. the three circles of radius <math>1</math> centered at
 +
<math>A, B, D</math> cover <math>\triangle ABD</math>) if an only if
 +
<math>a \le \cos \alpha + \sqrt{3} \sin \alpha</math>.
 +
 +
(Solution by pf02, September 2024)
 +
 +
 +
==Solution 3==
 +
 +
This solution is very similar to Solution 2, except that
 +
we choose another point instead of <math>F</math>.  This will in fact
 +
simplify the proof.  Start like in Solution 2.
 +
 +
We can assume <math>a > 1</math>.  Indeed, refer to Remark 2 above to
 +
see that if <math>a \le 1</math> there is nothing to prove.
 +
 +
Note that instead of the statement <math>S1</math> we can consider the
 +
following statement <math>S1'</math>:
 +
 +
<math>S1'</math>: the circles of radius <math>1</math> centered at <math>A, B, D</math> cover
 +
<math>\triangle ABD</math>.
 +
 +
This is equivalent to <math>S1</math> because of the symmetry between
 +
<math>\triangle ABD</math> and <math>\triangle BCD</math>.
 +
 +
Let <math>O</math> be the center of the circle circumscribed to
 +
<math>\triangle ABD</math>.  Let <math>M, N, P</math>, be the midpoints of
 +
<math>AB, AD, BD</math>. The three circles cover <math>\triangle ABD</math>
 +
if an only if <math>O</math> is inside the circle of radius 1
 +
centered at <math>D</math>.
 +
 +
[[File:Prob_1967_1_fig3.png|300px]]
 +
 +
This needs an explanation.  In fact, since <math>OA = OB = OD</math>,
 +
the point <math>O</math> is inside or on the circle centered at <math>D</math>
 +
if and only if <math>OD \le 1</math>, if and only if <math>O</math> is in or
 +
inside the circles centered at <math>A, B, D</math>.  Since
 +
<math>OP \perp BD</math>, we have <math>DP < OD, PB < OB</math>, so the triangles
 +
<math>\triangle OPD, \triangle OPB</math> are inside the circles
 +
centered at <math>D, B</math> respectively.  By drawing <math>OA, OM</math>
 +
we can easily verify that the whole triangle is inside
 +
the circles centered at <math>A, D, B</math>.
 +
 +
Note that in the above argument we used that <math>O</math> is inside
 +
<math>\triangle ABD</math>, which is true because the triangle is acute.
 +
 +
Denote <math>R</math> the radius of the circle circumscribed to
 +
<math>\triangle ABD</math>.  From the law of sines, we have
 +
<math>\frac{BD}{\sin \alpha} = 2R</math>, and from the law of
 +
cosines we have <math>BD^2 = 1 + a^2 -2a \cos \alpha</math>.
 +
So <math>R = OD \le 1</math> translates to
 +
<math>\frac{\sqrt{1 + a^2 -2a \cos \alpha}}{2 \sin \alpha} \le 1</math>
 +
 +
Since <math>\sin \alpha > 0</math>, we can simplify this inequality, and
 +
get
 +
 +
<math>a^2 - 2a \cos \alpha + (1 - 4 \sin^2 \alpha) \le 0</math>.
 +
 +
But this is exactly the inequality we encountered in Solution 2,
 +
so proving that this is equivalent to
 +
 +
<math>a \le \cos \alpha + \sqrt{3} \sin \alpha</math>
 +
 +
is identical to what has been done above.
 +
 +
(Solution based on an idea by feliz; see link in Remark 4, or below.)
 +
 +
 
-----
 
-----
Solution can also be found here [https://artofproblemsolving.com/community/c6h21154p137323]
+
 
 +
A solution can also be found here [https://artofproblemsolving.com/community/c6h21154p137323]
  
 
== See Also == {{IMO box|year=1967|before=First question|num-a=2}}
 
== See Also == {{IMO box|year=1967|before=First question|num-a=2}}

Latest revision as of 12:44, 19 September 2024

Let $ABCD$ be a parallelogram with side lengths $AB = a$, $AD = 1$, and with $\angle BAD = \alpha$. If $\Delta ABD$ is acute, prove that the four circles of radius $1$ with centers $A$, $B$, $C$, $D$ cover the parallelogram if and only if

$a\leq \cos \alpha+\sqrt{3}\sin \alpha$ $\ \ \ \ \ \ \ \ \ \ (1)$


Solution

To start our proof we draw a parallelogram with the requested sides. We notice that by drawing the circles with centers A, B, C, D that the length of $a$ must not exceed 2 (the radius for each circle) or the circles will not meet and thus not cover the parallelogram.

To prove our conjecture we draw a parallelogram with $a=2$ and draw a segment $DB$ so that $\angle ADB=90^{\circ}$

This is the parallelogram which we claim has the maximum length on $a$ and the highest value on any one angle.

We now have two triangles inside a parallelogram with lengths $1, 2$ and $x$, $x$ being segment $DB$. Using the Pythagorean theorem we conclude:

$1^2+x^2=2^2\\x=\sqrt{3}$

Using trigonometric functions we can compute:

$cos\alpha=\frac{1}{2}\\sin\alpha=\frac{\sqrt{3}}{2}$

Notice that by applying the $arcsine$ and $arccos$ functions, we can conclude that our angle $\alpha=60^{\circ}$

To conclude our proof we make sure that our values match the required values for maximum length of $a$

$a\leq\cos\alpha+\sqrt{3}\sin\alpha\\\\a\leq\frac{1}{2}+\sqrt{3}\cdot \frac{\sqrt{3}}{2}\\\\a\leq 2$

Notice that as $\angle\alpha$ decreases, the value of (1) increases beyond 2. We can prove this using the law of sines. Similarly as $\angle\alpha$ increases, the value of (1) decreases below 2, confirming that (1) is only implied when $\Delta ABD$ is acute.

--Bjarnidk 02:16, 17 May 2013 (EDT)


Remarks (added by pf02, September 2024)

$\mathbf{Remark\ 1}$. I am sorry to be so harshly critical, but the solution above is deeply flawed. Not only it has errors, but the logic is flawed.

It shows that when $a = 2, \alpha = \frac{\pi}{3}$ the parallelogram is covered by the circles of radius $1$ centered at $A, B, C, D$, and the inequality in the problem is true. (Even this is incomplete, while giving too many, unnecessary details.) (Note that this is not a case which satisfies the conditions of the problem since $\triangle ABD$ is right, not acute.)

In the last two lines it gives some reasoning about other values of $\alpha$ which is incomprehensible to this reader.

In one short sentence: this is not a solution.

$\mathbf{Remark\ 2}$. The problem itself is mildly flawed. To see this, denote $S1, S2$ the following two statements:

S1: The parallelogram $ABCD$ is covered by the four circles of radius $1$ centered at $A, B, C, D$.

S2: We have $a \le \cos \alpha + \sqrt{3} \sin \alpha$.

The problem says that if $\triangle ABD$ is acute, $S1$ and $S2$ are equivalent, i.e. they imply each other.

Notice that $S2$ can be rewritten as $a \le 2 \cos \left( \alpha - \frac{\sqrt{\pi}}{3} \right)$.

Now notice that if $a \le 1$ then S1 is obviously true. See the picture below:

Prob 1967 1 fig1.png

Also, notice that if $a \le 1$ and $\alpha \in \left( 0, \frac{\sqrt{\pi}}{2} \right)$ then $S2$ is true as well. Indeed $\left( \alpha - \frac{\sqrt{\pi}}{3} \right) \in \left( -\frac{\sqrt{\pi}}{3}, \frac{\sqrt{\pi}}{6} \right)$, so $\cos$ is $> \frac{1}{2}$ on this interval, so the right hand side of $S2$ is $> 1 \ge a$.

We see that if $a \le 1$ and $\triangle ABD$ is acute, both $S1$ and $S2$ are true. We can not say that one implies the other in the usual meaning of the word "imply": the two statements just happen to be both true.

If we take $a > 1$ then the problem is a genuine problem, and there is something to prove.

$\mathbf{Remark\ 3}$. In the proofs I give below, we will see where we need that $\triangle ABD$ is acute. We will see that $\alpha < \frac{\pi}{2}$ is needed for the technicalities of the proof. The fact that $\angle ADB$ is acute will be needed at one crucial point in the proof.

In fact, it is possible to modify $S2$ to a statement $S3$ similar to $S2$ so that $S1$ and $S3$ are equivalent without any assumption on $\alpha$. I will not go into this, I will just give a hint: Denote $\beta = \angle ABC$. If $\alpha$ is acute, $\beta$ is obtuse, and we can easily reformulate $S2$ in terms of $\beta$.

$\mathbf{Remark\ 4}$. Below, I will give two solutions. Solution 2 is one I carried out myself and relies on a straightforward computation. Solution 3 (it happens to be similar to Solution 2) is inspired by an idea by feliz shown on the web page https://artofproblemsolving.com/community/c6h21154p137323 The author calls their text a solution, but it is quite confused, so I would not call it a good solution. The idea though is good and nice, and it yields a nice solution.


Solution 2

We can assume $a > 1$. Indeed, refer to Remark 2 above to see that if $a \le 1$ there is nothing to prove.

Note that instead of the statement $S1$ we can consider the following statement $S1'$:

$S1'$: the circles of radius $1$ centered at $A, B, D$ cover $\triangle ABD$.

This is equivalent to $S1$ because of the symmetry between $\triangle ABD$ and $\triangle BCD$.

Let $F$ be the intersection above $AB$ of the circles of radius $1$ centered at $A, B$. The three circles cover $\triangle ABD$ if an only if $F$ is inside the circle of radius 1 centered at $D$.

Prob 1967 1 fig2.png

This needs an explanation: Let $H$ be the midpoint of $BD$, and consider $\triangle FHD$. All the vertices of this triangle are in the circle centered at $D$, so the whole triangle is inside this circle. It is obvious that $\triangle FHB$ is inside the circle centered at $B$, and that $\triangle FAD, \triangle FAB$ are inside the circles centered at $A, B$.

We will now show that $F$ is inside the circle of radius 1 centered at $D$ if an only if $DF \le 1$.

The plan is to calculate $DF$ in terms of $a, \alpha$ and impose this condition. Let $FG \perp AB$, $DE \perp AB$ and $FF' \parallel GE$. From the right triangle $\triangle AFG$ we have $FG = \sqrt{1 - \left( \frac{a}{2} \right)^2} = \frac{\sqrt{4 - a^2}}{2}$. From the right triangle $\triangle DFF'$ we have

$DF = \sqrt{(DF')^2 + (FF')^2} = \sqrt{(DE - FG)^2 + (AG - AE)^2} = \sqrt{\left( \sin \alpha - \frac{\sqrt{4 - a^2}}{2} \right)^2 + \left( \frac{a}{2} - \cos \alpha \right)^2}$

(Note that here we used the fact that $\alpha$ is acute. These equalities would look slightly differently otherwise.)

Now look at the condition $DF \le 1$, or equivalently $DF^2 \le 1$. Making all the computations and simplifications, we have $\sqrt{4 - a^2} \sin \alpha \ge 1 - a \cos \alpha$.

Now I would like to square both sides. In order to get an equivalent inequality, we need to know that $1 - a \cos \alpha \ge 0$. This follows from the fact that $\angle ADB$ is acute. Indeed, denote $\angle ADB = \beta$. From the law of sines in $\triangle ADB$ we have $\frac{1}{\sin \angle ABD} = \frac{a}{\sin \beta}$. Successively this becomes $\frac{1}{\sin (\pi - \alpha - \beta)} = \frac{a}{\sin \beta}$ or $\frac{1}{\sin (\alpha + \beta)} = \frac{a}{\sin \beta}$ or $(1 - a \cos \alpha) \sin \beta = a \sin \alpha \cos \beta$. From here we see that $\beta < \frac{\pi}{2}$ implies the right hand side is positive, so $1 - a \cos \alpha > 0$.

Going back to our inequality, we can square both sides, and after rearranging terms we get that $DF \le 1$ if and only if

$a^2 - 2a \cos \alpha + (1 - 4 \sin^2 \alpha) \le 0$.

View this as an equation of degree $2$ in $a$. The value of the polynomial in $a$ is $\le 0$ when $a$ is between its solutions, that is

$\cos \alpha - \sqrt{3} \sin \alpha \le a \le \cos \alpha + \sqrt{3} \sin \alpha$.

Note that $\cos \alpha - \sqrt{3} \sin \alpha = 2 \cos \left( \alpha + \frac{\pi}{3} \right)$. If $\alpha \in \left( 0, \frac{\pi}{2} \right)$ then $\left( \alpha + \frac{\pi}{3} \right) \in \left( \frac{\pi}{3}, \frac{5\pi}{6} \right)$, and it follows that $2 \cos \left( \alpha + \frac{\pi}{3} \right) \le 1$.

On the other hand, remember that we are in the case $a > 1$, so the left inequality is always true. It follows that

$DF \le 1$ (i.e. the three circles of radius $1$ centered at $A, B, D$ cover $\triangle ABD$) if an only if $a \le \cos \alpha + \sqrt{3} \sin \alpha$.

(Solution by pf02, September 2024)


Solution 3

This solution is very similar to Solution 2, except that we choose another point instead of $F$. This will in fact simplify the proof. Start like in Solution 2.

We can assume $a > 1$. Indeed, refer to Remark 2 above to see that if $a \le 1$ there is nothing to prove.

Note that instead of the statement $S1$ we can consider the following statement $S1'$:

$S1'$: the circles of radius $1$ centered at $A, B, D$ cover $\triangle ABD$.

This is equivalent to $S1$ because of the symmetry between $\triangle ABD$ and $\triangle BCD$.

Let $O$ be the center of the circle circumscribed to $\triangle ABD$. Let $M, N, P$, be the midpoints of $AB, AD, BD$. The three circles cover $\triangle ABD$ if an only if $O$ is inside the circle of radius 1 centered at $D$.

Prob 1967 1 fig3.png

This needs an explanation. In fact, since $OA = OB = OD$, the point $O$ is inside or on the circle centered at $D$ if and only if $OD \le 1$, if and only if $O$ is in or inside the circles centered at $A, B, D$. Since $OP \perp BD$, we have $DP < OD, PB < OB$, so the triangles $\triangle OPD, \triangle OPB$ are inside the circles centered at $D, B$ respectively. By drawing $OA, OM$ we can easily verify that the whole triangle is inside the circles centered at $A, D, B$.

Note that in the above argument we used that $O$ is inside $\triangle ABD$, which is true because the triangle is acute.

Denote $R$ the radius of the circle circumscribed to $\triangle ABD$. From the law of sines, we have $\frac{BD}{\sin \alpha} = 2R$, and from the law of cosines we have $BD^2 = 1 + a^2 -2a \cos \alpha$. So $R = OD \le 1$ translates to $\frac{\sqrt{1 + a^2 -2a \cos \alpha}}{2 \sin \alpha} \le 1$

Since $\sin \alpha > 0$, we can simplify this inequality, and get

$a^2 - 2a \cos \alpha + (1 - 4 \sin^2 \alpha) \le 0$.

But this is exactly the inequality we encountered in Solution 2, so proving that this is equivalent to

$a \le \cos \alpha + \sqrt{3} \sin \alpha$

is identical to what has been done above.

(Solution based on an idea by feliz; see link in Remark 4, or below.)



A solution can also be found here [1]

See Also

1967 IMO (Problems) • Resources
Preceded by
First question
1 2 3 4 5 6 Followed by
Problem 2
All IMO Problems and Solutions